Chapter 2 RPV Sol

You might also like

Download as pdf or txt
Download as pdf or txt
You are on page 1of 11

33. Let the larger number be a.

Using both the statements we have


Smaller number = 250 − a man’s age = 42 + 2 = 44 years
a2 − (250 − a)2 = 12500 ∴ Clearly man’s age < wife’s age + son’s age. Choice (C)
(a + 250 − a) (a − (250 − a)) = 12500
2a − 250 = 50 40. As statements Ι and ΙΙ individually do not relate to Uno and
Zen, they are not sufficient.
∴ a = 150 Choice (C)
Using both, we have Palio = 4/5 (Alto);
34. Let the number of marbles with Dinesh, Eswar, Ganesh Alto = Zen − 2
and Harish be d, e, g and h respectively. 4
Also Alto gives 45 × = 60 km
Given that d + e + g + h = 20 → (1) 3
1 1 ∴ Zen = 62 km
d= (e + g + h) = (120 − d)
2 2 ∴ Zen gives more mileage. Choice (C)
d = 40
1
Chapter – 2
similarly, e = (120 − e) e = 30 and (Ratio – Proportion – Variation)
3
1 Concept Review Questions
g= (120 − g) g = 24
4 Solutions for questions 1 to 30:
h = 120 − (d + e + g) = 120 − (30 + 40 + 24) = 26
Ans: (26) 9
1. Given a = b
8
35. Before doubling, the amounts with Bhavan, Chetan and
3b 3b 2
80 = = Choice (A)
Dinesh, each of them must have had = 40. 4a 9 3
2 4 b
8
∴ Amar must have then had 80 + 120 i.e. 200. Similarly
we can work out the amounts with each of them before the
other doubled the amounts. The results are summarized below. 2. Given 2.4p = 0.08q
2.4
q= p = 30p
Amar Bhavan Chetan Dinesh 0.08
Finally 80 80 80 80 q + p 31p 31
= = Choice (C)
Before Amar q − p 29p 29
200 40 40 40
doubles
Before Bhavan 3. Let a = 2x b = 3x
100 180 20 20 3a + 4b = 3 (2x) + 4 (3x) = 18x
doubles
Before Chetan 4a + 5b = 4 (2x) + 5 (3x) = 23x
50 90 170 10
doubles 3a + 4b 18
= Choice (B)
Before Dinesh 4a + 5b 23
25 45 85 165
doubles
4. a:b=2:3=4:6
Choice (D) a:c=2:3=6:9
∴a:b:c=4:6:9 Choice (A)
Solutions for questions 36 to 40:
5
5. p:q=5:6 p= q
x 3y 6
36. Statement Ι: + =1
2 4 4 4
q:r= q= r
Statement Ι is sufficient. 7 7
Statement ΙΙ: 9y + 6x = 12, we can see that this is 5 4 10
equivalent to 3y + 2x = 4. ∴ p= r = r
6 7 21
The value of 2x + 3y; value can be found, so statement
ΙΙ is sufficient. Choice (B) 10r 4
∴ p : q: r = : r :r = 10 : 12 : 21 Choice (A)
21 7
37. From statement ΙΙ, if the cost of 12 shirts is doubled, it will
be the same as 72 pants cost of 12 shirts is the same as a b c d 2 6 14 1 4
6. x x x = x x x = Choice (D)
the cost of 36 pants. So, only statement ΙΙ is sufficient and b c d e 3 7 25 2 25
the difference is zero. Choice (A)
7. Let P = 5x q = 4x
38. There are two possibilities 5x = a + b --- (1)
Ι. 21 attempts __ all correct. 4x = a − b--- (2)
ΙΙ. 25 attempts __ 22 correct. From (1) and (2),
Hence, the question cannot be answered even after using 2a = 9x and 2b = x
both the statements. Choice (D) ∴ a : b = 2a : 2b = 9 : 1 Choice (B)

39. From statement Ι, we have man’s age = wife’s age + 2 years 4


But to know whether man’s age = wife’s age + son’s age,
8. Number of sweets received by Sita = (35) = 20
7
we should know the ages of at least two of them. Ans: (20)
∴ Statement Ι alone is not sufficient.
From statement ΙΙ, we have 9. Let the numbers be 2x and 5x.
son’s present age = 12 years 2x + 4 4
=
wife’s present age = 42 years 5x + 4 9
But to know whether man’s age = wife’s age + son’s age or 18x + 36 = 20x + 16
not, we should know the relationship between the ages of x = 10
the man and wife or man and the son. 2x = 20 and 5x = 50
∴ Statement ΙΙ alone is not sufficient. ∴ The numbers are 20 and 50 Choice (D)
10. A : B : C = 7 : 5 : 4 s1 3
let A’s age = 7k B’s age = 5k ∴ < .
s2 4
c’s age = 4k = 32 k=8
= 7k + 5k + 4k = 16 x 8 = 128 years Ans: (128) s1 13
Only Choice (C), i.e. = satisfies this condition.
s2 20
11. Let the ages of the four members be 9k, 8k, 3k and
Choice (C)
2k years respectively.
9k + 8k + 3k + 2k 20. Let the present ages of the husband and the wife be
Average age = = 22
4 5x years and 4x years respectively.
k=4
∴ The age of the eldest family member (i) Ratio of their ages 20 years ago
= 9 x 4 = 36 Choice (A) 5 x − 20 5(x − 5 ) + 5 5
= = > (some positive quantity)
12. Let the number of marbles with A and B be 10x and 11x 4 x − 20 4(x − 5 ) 4
respectively 5
Total number of marbles = 21x ∴ the ratio must exceed . Only choice (D) satisfies
4
∴ the total number of marbles must be divisible by 21. Only
Choice (D) violates this condition. Choice (D) this condition Choice (D)

13. Let the ratio of the number of boys and girls in the class be (ii) Ratio of their ages 20 years hence
a : b. Number of boys and girls will be ak and bk. 5 x + 20 5(x + 5 ) − 5 5
Given ak + bk = 70 = = <
4 x + 20 4(x + 5 ) 4
70
K= . 5
a+b ∴ the ratio must be less than
4
Since k is an integer a + b must be a factor of 70; from the Only Choice (B) satisfies this condition. Choice (B)
options
The sum of the ratio in options (A), (B), (C) are factors of 21. Let the four numbers be x , 2x, 3x, 4x
70. But the sum of ratio 9 : 2 is not a factor of 70. The sum of squares = 480
∴ 9 : 2 is not a possible ratio. Choice (D) x² + (2x)² + (3x)² + (4x)² = 480 30x² = 480
14. Let the number of gents in the party be g. x² = 16
3 x=4
Number of ladies in the party = g ∴ The numbers are 4, 8, 12, 16 Choice (C)
4
Number of ladies in the party if 8 ladies join the party
q+r p+r p+ q
3 5 22. Given that = = =k
= g+8 = g g = 16 Ans: (16) p q r
4 4
q + r = pk, p + r = qk and p + q = rk
15. Let the present ages of the mother and son be m and s years q + r + p + r + p + q = k(p + q + r)
respectively (p + q + r) (2 − k) = 0
m:s=4:1 จ p + q + r ≠ 0,
m = 4k, s = k ∴2−k=0
after twelve years
k=2 Choice (D)
4k + 12 2
=
k + 12 1 23. x ∝ y² x = k x y²
k=6 4 = k x 122
∴ the present ages of the mother and son are 24 and k = 4/122
6 years respectively. Choice (C) When y = 18,
x = ky² x = 4/12² x 18 x 18 = 9 Choice (B)
16. Let the number of chocolates A and B be 3k, 4k
3k − 4 5 24. Area x (Diagonal)2
=
4k + 4 9 A x D2
∴ A had 3k = 24 chocolates initially. Ans: (24) A = KD2
A1 A 32 72
17. Let the numbers of students in A, B and C be 3x, 7x and 8x = 2 =
D21 D22 82 D 2 2
respectively.
If 10 students leave C and join B, C and B would have D2 = 12 units. Ans: (12)
8x − 10 students and 7x + 10 students respectively.
7x + 10 = 80 S1 D
25. = 1
x = 10 S2 D2
Total number of students = 18x = 180 Ans: (180) S1 = 10400, D1 = 26 and D2 = 24
SD
18. Let the incomes of A and B be 4x and 3x S 2 = 1 2 = 9600 Ans: (9600)
Their savings be 5y, 6y D1
Their expenditures are (4x – 5y) and 3x – 6y
4x – 5y = ¾ (4x) x = 5y 26. As the distance to be covered is constant, S1 T1 = S2 T2
∴ the expenditure of A = 4x – 5y = 4(5y) – 5y = 15y S1 = 50, T1 = 6 and S2 = 100
∴ the expenditure of B = 3x – 6y = 3(5y) – 6y = 9y ST
T2 = 1 1 = 3 Choice (B)
∴ the ratio of the expenditure of A and B is S2
= 15y : 9y = 5 : 3 Choice (B)
27. P = k (Q + R) where k is a proportionality constant.
s1 e s s +e e P1 = k (Q1 + R1)
19. If < 1 then 1 < 1 1 < 1
s2 e2 s2 s 2 + e2 e 2 P2 = K (Q1 − 1 + R1 − 1) = k (Q1 + R1 − 2) = P1 − 2k
e1 + s1 3 e 4 As k is unknown, the change in P cannot be found.
In the given problem, = and 1 = Choice (D)
e2 + s2 4 e2 5
28. From the given information, A varies directly with the 5. 3x − 4y + 2z = 0 ---(Ι)
product of B and C. 4x − 2y − z = 0 -----(ΙΙ)
A BC From (II)
∴ 1 = 1 1
A 2 B 2C2 z = 4x − 2y
Substituting z = 4x – 2y in I, we get
A1 = 6000, B1 = 20, C2 = 30, B2 = 40 and C2 = 60
3x − 4y + 8x − 4y = 0
AB C
A 2 = 1 2 2 = 24000 Ans: (24000) 11x = 8y
B1C1 Again, z = 4x − 2y
multiply by 4 on both sides we get
29. A α B 4z = 16x − 8y; substituting 8y = 11x
A α 1/C 4z = 16x − 11x = 5x
A α B/C 5 11
x:z:y=x : x: x = 8 : 10 : 11 Choice (A)
A1C1 A 2C2 4 8
=
B1 B2
p−x 1 3p − q
6. = ;x= Choice (B)
20 × 30 A 2 × 42 q+ x 3 4
=
65 84
7. Two numbers are in the ratio 4 : 7
A2 = 24 Choice (A)
Let the numbers be 4k and 7k
Let the number added to each be L.
30. From the given information, X varies inversely with the
Given that, (4k + L) + (7k + L) = 75
product of Y and Z.
11k + 2L = 75 --------- (1)
X Y Z
∴ 1 = 2 2 4k + L 8
X2 Y1 Z1 It is also given that, =
7k + L 17
X1 = 30, Y1 = 8, Z1 = 7, 12k + 9L = 0 ----------- (2)
Y2 = 16 and Z2 = 21 −9 L −3
X YZ k= = L
X2 = 1 1 1 = 5 Ans: (5) 12 4
Y2 Z 2 Substituting the value of k in (1) we get L = –12

Exercise – 2(a) Alternate method:


Ratio of the resulting numbers = 8 : 17
Solutions for questions 1 to 30: Sum of the resulting numbers = 75
Hence, the numbers obtained after addition are
1. a/b = 3/7 (8/25) x 75 = 24 and (17/25) x 75 = 51
i.e., 4k + L = 24 and 7k + L = 51
a 12 On subtraction,
4 +5 +5
4a + 5b b 47
= = 7 = 7k − 4k = 51 − 24 3k = 27, k = 9 and L = −12
2a + 2b a 6 20 Ans: (–12)
2 +2 +2
b 7
8. Let the amounts initially with Mohan and Sohan be 8x and
5x respectively. Let us say Mohan spends y each day.
Alternate method:
Substituting the values of a and b as 3k and 7k respectively 8 x − y(9) 10
= x = 3y
in
4a + 5b
, 5 x + (y / 6 )(9 ) 11
2a + 2b Let us say Mohan and Sohan have amounts which are in
12k + 35k 47k 47 the ratio 18 : 35 after t more days.
we get = = Choice (A)
6k + 14k 20k 20 8 x − t.y 18 8(3 y ) − t.y 18
= i.e. =
5 x + t.y / 6 35 5(3 y ) + t.y / 6 35
2. a : b = 2 : 3 = (2 × 4) : (3 × 4) = 8 : 12
b : c = 4 : 3 = (4 × 3) : (3 × 3) = 12 : 9 t = 15
The ratio of the amounts with them would be 18 : 35 after
9 9 27 6 more days. Ans: (6)
c:d=2:3= 2× : 3× =9:
2 2 2
9. Let the present age of Kishore be x years,
27 Vipin's present age is 2(x – 1) years
a : b : c : d = 8 : 12 : 9 : = 16 : 24 : 18 : 27 Given the ratio of the sum of their present ages to the
2
difference of their present ages, is 19 : 5
Choice (B)
3 x − 2 19
i.e. = ∴x=7
3. Let a, b and c be the weights of the Bimal, Basu and Bali x−2 5
respectively. sum of present ages = (k + v) = 19 years Ans: (19)
a/b = 2/3 = 8/12; c/b = 3/4 = 9/12; a : b : c = 8 : 12 : 9
Given the sum of the weights (8 + 12 + 9 = 29 parts) is 203 kg 10. Ratio of prices = 5 : 8 : 13
i.e. one part is 7 kg. So Basu’s weight i.e. 12 parts is Ratio of number of balls = 5 : 4 : 3
12 x 7 = 84 kg Ans: (84) Ratio of amounts spent = (5 × 5) : (8 × 4) : (13 × 3)
= 25 : 32 : 39
4. Number of boys = 7x Total number of parts of the ratio=25 + 32 + 39 = 96
Number of girls = 3x Total amount spent, as per data = 768
7x 2 Value of each part of the amounts' ratio=768/96 = 8
= Amount spent on costliest variety
3 x + 15 3
= Value of 39 parts = 39 × 8
21x = 6x + 30
Number of costliest variety balls Paul purchased is
15x = 30 , x = 2
= (39 × 8)/13 = 24 Choice (D)
∴ Number of girls = 6 Choice (D)
11. ( a + c ) + (c + e ) + ( a + e ) 2(a + c + e ) a + c + e
Brother 1 2 3 4 5 6 7 8 9 k= = =
(b + d) + ( d + f ) + (b + f ) 2(b + d + f ) b+d+ f
Amount
with the x 2x + 2 3x + 3 4x + 4 ... ... ... ... 9x + 9 a+c a+c +e (a + c + e ) − ( a + c ) e
Now = =k = = =k
brother b+ d b+d+ f (b + d + f ) − (b + d) f
c +e a+c +e
9x + 9 Similarly = =k
= 10, x = 9; d+ f b+ d+ f
x a + c + e ) − (c + e )
th
a
5 brother gets (5x + 5) = 45 + 5 = 50 Choice (D) k= = and lastly,
(b + d + f ) − ( d + f ) b
12. Let p. q, r and s represent the apples received by Karan, ( a + c + e ) − (a + e ) c
Kiran, Kumar and Khanna. k = = , hence all the choices (A), (B)
(b + d + f ) − (b + f ) d
Given q : (p + s) = 1 : 2,
and (C) are true. Choice (D)
q : (p + s) : (q + p + s) = 1 : 2 : (1 + 2) = 1 : 2 : 3 ------ (1)
r : (p + q + s) = 2 : 5-------- (2)
s = 2 + q ------- (3) 15 12
17. In the two cases, the greatest parts are N and N
and p : r = 1 : 2 or r : p = 2 : 1 ------- (4) 42 42
From (2) × 3 and (1) × 5, we have 3N
r : (p + q + s) : (p + s) : q = 6 : 15 : 10 : 5 Their difference = =6 N = 84 Ans: (84)
42
Combining this with (4) × 3, we have
r : p : (p + q + s) : (p + s) : q = 6 : 3 : 15 : 10 : 5 18. Let the two numbers be “a” and “b”.
r : p : q : (p + q + s) : (p + s) = 6 : 3 : 5 : 15 : 10
ab = 9
r : p : q : (p + q + s − p + q) : (p + q + s) : (p + s)
Squaring on both sides
= 6 : 3 : 5 : (15 − 3 + 5 ) : 15 : 10 ab = 81 ––––– (1)
b2 = 243a --- (2)
r : p : q : s : (p + q + s) : (p + s) = 6 : 3 : 5 : 7 : 15 : 10 multiplying both the sides with b, we get
From (3), s − q = 2 and this is satisfied by the number of b3 = 243 × 81
parts of s and q in the above ratio. b = (35 × 34)1/3 = 33 = 27 a=3
The number of parts in the above ratio are the actual The larger of the two numbers = 27 Choice (A)
values.
p = 3, q = 5, r = 6, s = 7; i.e., p + q + r + s = 21 19. Let P = ab and Q = cd Ps = a + b and Qs = c + d
Choice (A)
10a + b 10c + d
=
a+b c +d
9 th
13. David received of the number of toffees distributed by (10a+b) (c+d) = (10c+d) (a + b)
17 10ac + bc + 10ad + bd = 10ac + ad + 10bc + bd
1 th ad = bc ……(1)
Alok. Amitha received of the number of toffees Let a = 1, b = 0, c = 2 d = 0
16
distributed by David. This would correspond to the minimum value of Ps+Qs,
which is 3. Choice (D)
1 9
T = 18
16 17 d1 d2 48 d2
T = 544 Ans: (544) 20. = ; = d2 = 147 Choice (D)
t12 t 22 16 49
14. Quantity of diesel purchased = x lts.
∴ Cost per liter of diesel = 510/x 21. Let the sum of money be `x
510 × 3
The sum is sufficient to pay A’s wages for 55 days and B’s
Cost per liter of kerosene = = 306/x wages for 66 days.
x×5
x x
Since equal volumes are purchased, amount spent on Daily wage of A = and daily wage of B =
55 66
306 Number of days that the sum is sufficient to pay the wages
kerosene = × x = 306 Choice (B)
x x
of both workers = = 30 Ans: (30)
x x
15. Amount spent on kerosene = x +
Amount spent on petrol = 5x 55 66
∴ Cost involved = 6x ---------- (1)
Ratio of prices per liter of Petrol, diesel and kerosene is 22. V α r2h; V = k.r2h; V1/( r12h1 )
15 : 5 : 3 2
When equal volumes of all are purchased, the amounts are = V2 / ( r2 h 2 ) = k
also in the ratio 15 : 5 : 3. Hence, 66/(9 × 7) = 308/(6r²); r=7 Choice (C)
Amount spent on diesel, in this case = 510
Total amount spent = (23 × 510)5 = 23 × 102----- (2) 23. Let K, M and S be the kinetic energy, mass and speed of a
From (1) and (2), 6x = 2346 body respectively.
x = 391 Given:
∴ Amount spent on petrol = 391 × 5 = 1955 K ∝ S² (when M is kept constant)
Choice (C) and K ∝ M (when S is kept constant)
K ∝ MS² K = CMS²
a+c c +e a+e where C is the constant of proportionality.
16. = = =k Given that when M = 2 kg, S = 10 m/s, K = 100 joules
b + d d+ f b+ f
100 = C x 2 x 10² C = 1/2 K = 1/2 MS²
Using the relationship each ratio When M = 20 kg and S = 1 m/s
K = 1/2 x 20 x 1² = 10
sum of numerators ∴ A body of mass 20 kg moving with a speed of 1 m/s has
=k=
sum of deno min ators a kinetic energy of 10 joules. Choice (C)
24. Let the fixed income and the royalty that he gets be
27. v = 40 – k 3 n , where v is the speed when n wagons are
denoted by F and R respectively.
Total income (T) = F + kB (∴R α B). attached.
46000 = F + 2000k If n = 27, v = 34
66000 = F + 3000k 3
34 = 40 – k 27
Solving these, k = 20. F = 6000
Total income on setting 6000 books = F + 6000k 3
34 = 40 – k(3) k = 2 ∴ v = 40 – 2 n
F + 6000k 6000 + 6000( 20) Minimum speed required = 30
Income per book = = = 21.
6000 6000 3
∴ 30 = 40 – 2 n ;
Ans: (21)
3
5= n
25. A α (B + C) A = k1 (B + C) n = 125
Bαx B = k2 x So, a maximum of 125 wagons can be attached.
1 k Choice (B)
Cα C= 3
x x
28. Let the initial weight of the stone (before breaking) be 6 w.
k3 Weights of the broken stones are w, 2w and 3w
∴ A = k1 k 2 x +
x respectively.
Initial value of the stone (in ) = 10, 872
As A = 3, when x = 1,
10,872 α 36 w2
3 = k1 [k 2 + k 3 ] ---- (Ι)
10,872 = 36 w2 k
As per data, A = 3, when x = 2 Total value of the pieces of the stone
= (12 + 22 + 32) w2 k = 14 w2 k
k3 Loss in the value = 22 w2 k
∴ 3 = k1 2k 2 + ----- (ΙΙ)
2
10872
= 22 × = 22 × 302 = 6644
(Ι) – (ΙΙ) gives 36
k k k3 Choice (C)
− k1 k2 + 1 3 = 0 k1 − k2 = 0
2 2
29. distance = d weight = w
k1 = 0 Given d α (1/w)
k dw = k (constant)
or k2 = 3 are the possible cases Let weights of 3 pieces be x, 3x, 2x
2
k k k 11 k
k1 cannot be 0 (as A can’t be 0). ∴ Sum of distances = + + = = 22
x 3 x 2x 6 x
k
Hence k2 = 3 k = 12x
2
Weight of unbroken stone is
From (Ι)
w = 6x = k/2 as d. w = k
3k1 k 3 d x k/2 = k d=2m Ans: (2)
3=
2
k1 k3 = 2. k1 k2 = 3 −2 = 1. 30. Fixed charge = x, unit rate = y, x and y are in Rupees.
5/4(x + 100y) = x + 200y
k1k 3 2 5x + 500y = 4x + 800y -------- (1)
Hence A = k1k2 x + =x +
x x x + 50y = 700 ----------------(2) (data)
When x = 4, y = 2, x = 600
Choice (A)
2 9
A=4+ = = 4.5 Ans: (4.5)
4 2 Exercise – 2(b)

26. The quantity of the balance of food after the transfer is such Solutions for questions 1 to 40:
that
(900 − 300) = 600 soldiers, consumed at the rate of 1. Let a + b − c = 5x
3 kg/day/soldier, for 25 days ------- (1) b + c − a = 6x
If the soldiers were not transferred, 900 soldiers would a + c − b = 7x
have consumed it at the rate of 2.5kg/day/soldier, the same (a + b − c) + (b + c − a) = 11x
food. ---------- (2) ∴ 2b = 11x
The data can be tabulated as: (a + b − c) + (a + c − b) = 12x
∴ 2a = 12x
Soldiers Consumption rate Number of days (b + c − a) + (a + c − b) = 13x
600 3.0 25
∴2c = 13x
900 2.5 How many?
∴ a : b : c = 2a : 2b : 2c = 12 : 11 : 13
Choice (B)
Number of soldiers and the number of days for which
food lasts are inversely proportional. The number of
soldiers increased; hence, number of days decreases. 20p 2 − 40pq
2. Given, = 20 20p2 − 60pq − 80q2 = 0
Hence multiplication factor is (600/900). pq + 4q2
Consumption rate and number of days are also inversely
proportional. Hence, multiplication factor is 3.0/2.5 p2 − 3pq − 4q2 = 0
Applying the above rates of variation, the number of days (p − 4q) (p + q) = 0
= 25 x (600/900) x (3.0/2.5) = 20 days
The initial stock was to last for 30 days. p
∴ = 4 or − 1
Soldiers were transferred after 30 − 20 = 10 days q
Ans: (10) ∴Choice (C) is possible. Choice (C)
3. We will find it convenient to change the notation slightly. Number of girls who are day scholars and travel to school
Let A : B = 3 : 4 and c : d = 2 : 3 1 2
Let A = 3x ∴ B = 4x by bus = (2y ) = 2 x
2 3 3
Let c = 2y ∴ d = 3y
1 2
A 3 c 2 + B 3 d2 x+ x
To find E = Required part = 12 3 =1 Choice (C)
AB 2 d2 + A 2Bcd 3x 4
As each term has 3 upper case letters and 2 lower case,
12. The data is tabulated below.
E is homogeneous and we can evaluate it.

3 3 ( 2 2 ) + 4 3 (32) x3 y2 White Black Green Total


E=
2 2 2
3( 4 )(3 ) + 3 ( 4)(2)(3) x y 3 2 3 4
3 5
3 + 42 19 9 12 20 41
= = Choice (D)
3( 4 ) + 6 18
The total number of white and green balls is a multiple of
4. Let the scores of Ajay be a and v respectively. 29. Among the choices, only 58 is a multiple of 29.
a + 2v = 310 ……(1) Choice (C)
v + 2a = 290 ……(2)
Solving (1) and (2) 13. Let the number of coins with Amar, Bhavan and Chetan be
a = 90 and v = 110 A, B and C respectively.
∴ a : v = 9 : 11 Choice (A)
The data is tabulated below
5. Let the first and second parts be a and b. A B C AB AC BC 2(ABC)
3 5 4 12
3a 25 a 25
Given, = = ∴ A = 6 − 4 = 2, B = 6 − 5 = 1 and C = 6 − 3 = 3
2b 4 b 6
∴B:C=1:3 Choice (D)
25
a= (93) = 75 Ans: (75)
31 14. Let the price of tea last year be 5x per kg. Let the price of
coffee last year be 7y per kg.
6. Let the fraction be x/y 6
x−3 1 (5x )+ 8 (7y )= 48
= 2x – y = 11 → (1) 5 7
y+5 2
20 3 3
x+2 5x = (7y ) y= x 6x = x = 48
= 1; x – y = –2 → (2) 21 4 4
y
x=4 5x = 20 Choice (B)
Subtracting (2) from (1), we get x = 13; y = 15
∴ Fraction = 13/15 Choice (C)
2
15. Rohan was supposed to get th of the total amount.
7. Let the number of students in three classes A, B and C be 9
3x, 7x and 8x respectively. 1 1 1
But the actual ratio of division was : : = 6: 4: 3 .
7 x + 10 8 2 3 4
Given, = x = 10
8 x − 10 7 6
∴ he got th of the total amount.
Total = 18x = 180 Ans: (180) 13
8. Let the numbers of sweets received by Ram, Shyam and 2 6
Tarun be r, s and t. As ≈ 0.2 and > 0.4, Rohan gained. He gained by
9 13
r 5 s 2 4
= and = = 6 2
s 4 t 3 6 − 117 = 28 Choice (C)
3 9
6
∴r:s:t=5:4:6 t= (60) = 24 Ans: (24)
15 16. Let the quantities of milk be 3xml, 4x ml, 5x ml and 6x ml
and 7x ml.
9. Let the 3 parts be x, y, z. Let the capacity of each vessel be 100 ml.
∴ x + y + z = 66 --------- (Ι) Total capacity = 500 ml
z=x+y Substituting z in (Ι) 2x + 2y = 66 60
x + y = 33 --------- (ΙΙ) 3x + 4x + 5x + 6x + 7x = (500) x = 12
100
Given y = 2x – 3 Substituting y in (ΙΙ),
∴ 3x = 36, 4x = 48, 5x = 60, 6x = 72, 7x = 84
we get 3x – 3 = 33 x = 36/3 = 12
The last 3 vessels contain at least 50% milk.
∴ y = 21 and z = 33 Ans: (3)
Ratio of x, y and z is = 12 : 21 : 33 = 4 : 7 : 11
Choice (D)
17. Let the monthly income of Ashok be 3x. Let the expenditure
of Ashok be 4y.
10. Number of weighing stones of 500 gms = 5000/500 = 10
Monthly income of Bala = 4x
∴ Number of 100 gm weights = (3/5) × 10 = 6
Expenditure of Bala = 5y
Choice (A)
Savings of Ashok and Bala are (3x − 4y) and (4x − 5y)
respectively.
11. Let the number of boys in the class be x.
Number of girls in the class = 2x. 1
y
Number of day scholars who are boys and girls travel to 3x − 4y 3 4 3
Ratio = = − which is less than . Only
1 1 1 4x − 5y 4 4x − 5y 4
school by bus = x = x. Choice (B) violates this condition. Choice (B)
4 3 12
18. Data can be tabulated as follows: 23. Total number of questions is 22.
Number of English questions = (7 × 22)/11 = 14
Manoj Shiva Time taken for English questions
1. Income Ratio 3 : 4 = (60 × 7)/15 = 28 minutes
2. Savings Ratio 2 : 3 Number of questions that can be answered in 18 minutes
3. Spending = 18/(28/14) = 9 Choice (D)
(Expenditure) - -
m p
Manoj's expenditure is 2/3rds
of his income. Let Manoj's 24. When two ratios like and are equal, each of them is
monthly income be 36. (an arbitrary number is chosen in n q
such a way that no fractions are encountered in the m−p
calculation) equal to , provided n ≠ q.
n−q
Hence, Manoj's expenditure is 36 x (2/3) = 24 ---- (1)
Hence, Manoj's savings is 36 − 24 = 12
∴k=
(
(a 2 + c 2 ) − b 2 + c 2)=a+b Choice (C)
Ratio of savings of Manoj and Shiva = 2 : 3 (a + c ) − (b + c )
Hence, Shiva's savings = 3 x (12/2) = 18 ------- (2)
Income Ratio = 3 : 4
As Manoj's income = 36, Shiva's shall be p + q q+r p+r
25. As = =
4 x (36/3) = 48------------- (3) r p q
Shiva's expenditure = Income – Savings p + q+ q+r +p +r
= 48 − 18 = 30 --------- (4) Each of them equals = =2
r +p+ q
From (1) and (4); Ratio of expenditures of Manoj and Shiva
is 24 : 30 = 4 : 5 Choice (B) If p + q + r 0, then k = 2
p+q
If p + q = − r, then =−1
19. Let the numbers satisfying the condition be denoted by xyz. r
z = x + y, z − y = y − x z = 2y − x = x + y ∴ k = −1
y = 2x Sum of all the possible values of k is 1. Choice (A)
z = 3x
∴x:y:z=1:2:3
2x 2 − 4 x + 3 2x 2 − 3 x + 5
∴ (x, y, z) can be (1, 2, 3), (2, 4, 6) or (3, 6, 9) 26. =
Choice (C) 4x − 3 3x − 5
By componendo and dividendo
20. Let the present ages of the mother, her husband and her
daughter be x years, y years and z years respectively. ( 2 x 2 − 4 x + 3 ) + ( 4 x − 3 ) ( 2 x 2 − 3 x + 5 ) + (3 x − 5 )
=
x + z = 60 4x − 3 3x − 5
The mother would attain her husband’s age after y − x
years. 2x 2 2x 2
y+y−x = x = 0 or 4x − 3 = 3x − 5
∴ =2 4x − 3 3x − 5
z+y− x
x = − 2 or 0 Choice (D)
x = 2z
From (1), z = 20 Ans: (20) 27. (10a + b) (10c + d) = (10b + a) (10d + c)
100ac + 10bc + 10ad + bd
21. = 100bd + 10ad + 10bc + ac
ac = bd
Larger plot Smaller plot Combined a d a b
= or = Choice (A)
b c d c
Area M 8x 29y – 8x 29y
Area B 9x 33y – 9x 33y 28. Let the lengths of the larger and the smaller parts be x units
and y units respectively.
29 y − 8 x 13 x+y y x y
=6 + 1− 6 = 0
= x = 2y. y x y x
33y − 9x 15
2
Ratio of the area under maize cultivation in the larger plot x x x x
+ −6=0 +3 −2 =0
and that under barley cultivation in the smaller plot y y y y
8x 16 y 16
= = = Choice (B) x x
33 y − 9 x 33 y − 18 y 15 As + 3 > 0, = 2 Choice (A)
y y
22.
29. Let the numbers be a and b
Number of Time per a:9=9:b ab …….(1)
Total time a : b = b : 6561 b2 = 6561a …….(2)
questions question
Solving (1) and (2)
Mathematics a 2x 8y a = 1 and b = 81 Ans: (81)
English b x 7y
30. Let Ahmed’s age = x, Ahmed’s brother’s age = a
Let Mohammed’s age = y, Mohammed’s brother’s age = b
Given 15y = 60 x + y a+b
y = 4∴ 8y = 32 min 7y = 28 min x/y = a/b; = (using compoendo and dividendo)
x − y a−b
(a ) ( 2x ) 8
= [a/b = 16/28 = 4/7] x+y x−y 1 a+b
(b ) ( x ) 7 = = as per data∴ =2
a+b a −b 2 x+ y
∴ Ratio of the number of English questions to the number Choice (A)
of Maths = b/a = 7/4 Choice (B)
31. x = k (y2 + 4) P1V1 P2 V2
39 = k(13) k=3 36. = ;
T1 T2
60 = 3(y2 + 4)
y2 = 20 – 4 = 16, y = ± 4 (but, from the condition y > 0 xy (3 x ) ( 2 y )
= , T2 = 600 K
y = 4) Ans: (4) 100 T2

32. Let the three-digit numbers satisfying the given conditions ∴ T2 – T1 = 500 K Ans: (500)
be denoted by abc
Q
a+c 37. P α , other factors being constant where Q and V are
b= ……(1) V
2 mass and volume of the gas.
Let the volumes of three chambers be V1, V2, V3
ab bc respectively.
=
a +b b+c V1 V2 V3
10a + b 10b + c Let pressures in 3 chambers be P1, P2 and P3
=
a+b b+c P1V1 = P2 V2 = P3 V3 ----(1)
[Since PV = KQ and as Q is same for all chambers, KQ is
(10a + b) (b + c) = (10b + c) (a + b) constant]
10ab + b2 + 10ac + bc = 10ab + ac + 10b2 + bc V1 : V2 : V3 = 1 : 2 : 3
ac = b2 So, P1 = 2P2 = 3P3
2
a+c 6 6 6
From (1), ac = ∴, P1 P2 : P3 = : :
2 1 2 3
(a − c)2 = 0 P1 : P2 : P3 = 6 : 3 : 2 Choice (C)
∴a=c
From (1), b = c 38. H = K T − 20 where K is a constant.
∴a=b=c H 16
∴ Nine numbers satisfy these conditions. Choice (C) K= = =4
T − 20 36 − 20
33. Let the required distance be x m.
D = kt2 where k is a constant. 20 = 4 T − 20 T = 45 Choice (B)

D 500 39. Let the number of members be x.


k= = =5
t2 10 2 T=F+V
(110)(50) = F + 50K ….(1)
x = Distance the body falls in 9 seconds, Distance the body (80) (80) = F + 80K ….(2)
falls in 8 seconds = 5(92) − 5(82) = 85 m Ans: (85) In (1) and (2) K is a constant.
Solving (1) and (2)
34. Let Q = A + B y + C(3 y ) F = 4000 and K = 30
130x = 4000 + 30x
Where A, B and C are constants. x = 40 Ans: (40)
60 = A + B + C B + C = 60 − A
40. Constant part Variable rate Units Total
230 − A
230 = A + 8B + 4C 2B + C = x (Patna) y 100 x+100y = 500
4 5x + 900y
5/3x (Bangalore) y 300 = 1100
729 − A 3
729 = A + 27B + 9C 3B + C = Solving the above two equation, we get y = 2; x = 300
9
5
2B + C − (B + C) = 3B + C − (2B + C) x = 5/3 × 300 = 500
3
230 − A 660 − A 230 − A
∴ − (60 − A )= − Required difference = 500 − 300 = 200 Choice (A)
4 9 4
Solutions for questions 41 to 45:
Among the choices given, only Choice (C) satisfies the
equation above. Ans: (30) 41. Let 1 Rupee coins, be x; 50 paise coins be y and 25 paise
coins be z.
35. Let cost of supply = s From statement Ι, x + y + z = 20.
Price per unit = p From statement ΙΙ, x = 6 and y : z = 6 : 1.
s = k 1 + k 2 p + k 3 p2 Combining statements Ι and ΙΙ, we get x = 6, y = 12, z = 2.
Given, s = 9, when p = 1 Choice (C)
9 = k1 + k2 + k3 ----(I)
s = 24, when p = 2 42. Let the number of technical staff be 3x and non-technical
24 = k1 + 2k2 + 4k3 ---(II) staff be 10x.
s = 47 when p = 3 From statement Ι,
47 = k1 + 3k2 + 9k3 ----(III) 80 40
(II) – (I) gives 15 = k2 + 3k3 ----(IV) Total number of graduates = (3x) + (10x)
(III) – (II) gives 100 100
23 = k2 + 5k3 ---(V) = 2.4x + 4x = 6.4x
(IV) – (V) gives 6. 4 x
Percentage of graduates = × 100 = 64%
8 = 2k3 10 x
k3 = 4 So statement Ι alone is sufficient.
k2 = 15 − 12 = 3 From statement ΙΙ,
k1 = 9 − 4 − 3 = 2.
32 x
Hence, when p = 4 The required ratio = × 100%
s = k1 + 4k2 + 16k3 = 2 + 12 + 64 = 78 Choice (D) 65 x
So statement ΙΙ alone is also sufficient. Choice (B)
43. Let x be the number of boys and y be the number of girls A A
x 5 8. A% of + % of
From statement Ι, = 4x = 5y B B
y 4 A
x−4 A A
From statement ΙΙ, =1 x=y+4 A= + B (A )
y 100 B 100
Combining both the statements, we get the value of y which A2 A2 A2
is the number of girls. Choice (C) = + =
100 B 100 B 50 B
44. From statement Ι, we don’t know what percentage of boys A C A AC
C% of = =
and what percentage of girls are intelligent, so we can’t B 100 B 100B
answer the question.
A2 AC
From statement ΙΙ, =
50B 100B
20% of boys and 20% of girls are adults so (100 – 20)%
= 80% of students are not intelligent, whatever be the ratio A2
of boys and girls. Choice (A) 50B
C= =2A Ans: (2)
A
1 100B
45. From statement Ι, we have x = kg (assuming total as
8
a ab
1 kg) 9. a% of b = (b ) =
100 100
3 3 1
Also B = kg i.e., × kg b ab
16 2 8 b% of a = (a) =
100 100
As we do not know about others, we cannot find who
received least from this statement alone. 331 / 3
From statement ΙΙ, we have
331/3% of (a + b) = (a + b)
100
3 1 1
A= kg; Y = kg = (a + b)
8 8 3
Again statement ΙΙ alone is not sufficient, as we do not
ab ab 1
know about others. + = (a + b)
Using both, we know about four persons out of five. We can 100 100 3
find how much Z received and hence, also the one who 2ab 1
received the largest part. Choice (C) = (a + b)
100 3
Chapter – 3 6 a+b
=
(Percentages – Profit & Loss – Partnerships) 100 ab
6 1 1
Concept Review Questions = +
100 a b
Solutions for questions 1 to 50: 1 1
6% = + %. Choice (A)
a b
40 x 2000
1. × 50 = × 80 x= = 25
100 100 80 10. Let the value of the property be x
Choice (B) The value of the part that Ganesh owns (in )
1
83
200 3 5
2. × 100 = 250% Ans 250 = ×x= x
80 100 6
3 5
Three fourths of this part is worth x
3. Let the number be x. 4 6
32% of x = 256
5
32 i.e., x
x = 256 8
100
x = 800 Choice (B) 5
x = 5, x = 8 Ans: (8)
8
4. 60% of x = 60 + 60% of 60
60 11. Let the total marks be x
= 60 + (60) = 60 + 36 = 96. Choice (C) 80% of x = 720
100
720 ×100
x= = 900 marks
508 80
5. 1013/5% = % = 508/500 = 127/125 Choice (D) 90 × 900
5 90% of x = = 810 marks
100
6. Let the number be x. Alternate method:
60 70 If 80% of x = 720
(x ) = 1680 90% of x = 90/80 x 720 = 810 Choice (C)
100 100
42 12. Let the maximum mark in the test be M.
x = 1680 x = 4000. Ans (4000) 35
100 Pass mark = M
100
7. 40% of y = 75 + 20% of 1500 The student failed by 15 marks.
40 20 ∴ Pass mark R = 230 + 15 = 245
y = 75 + (1500)
100 100 35
M = 245 M = 700 Choice (A)
0.4 y = 375 y = 937.5. Choice (C) 100
1 Alternate method:
25 − 17
13. Reduction in the tax (in ) = 2 (9200) Any quantity which decreases by x% must be increased by
100 100 x
= 690 Ans: (690) % to become its original value
100 − x
14. Let the number of votes polled be x. 100 x
As x = 20, = 25 Choice (A)
70 100 − x
Number of votes secured by Ashok = x = 0.7x
100
He won by a majority of 0.4x votes. 25. Let the original price be x
0.4x = 168 x = 420 Choice (B) Increase in the price = 0⋅2x
Price after the increase = 1·2x
15. Let the original price be x 0 ⋅ 2x
Increase = 0⋅3x % decrease = (100 ) = 16 2 %
1 ⋅ 2x 3
x + 0⋅3x = 260 x = 200 Ans: (200)
Alternate method:
16. Profit = 300.
Any quantity which increases by x% must be decreased by
300
Profit percentage = (100 )% = 50% Choice (A) 100 x
% to become its original value
600
100 + x
17. Ratio of profits of P and Q = Ratio of the investments of 100 x 2
P and Q = 1: 2 As x = 20, = 16 % Choice (C)
100 − x 3
2
Q’s profit = (9600 ) = 6400 Ans: (6400)
3 5 2
26. Ratio = :
6 3
18. Let the original price be x.
Let the first number be 5x,
30
x− x = 63 x = 90 Choice (C) Second number is 4x
100
5x − 4x
% less = × 100
19. Let the marks of Mohan and Sohan be m and s 5x
respectively. = 20% Choice (A)
25 5
m = s 1+ = s. 27. Let the price of fan B initially be x
100 4
Price of fan A initially = 2x
m = minimum mark required to pass i.e. pass mark = 35.
5 10
s = 35 Increase in the price of A = (2x ) = 0 ⋅ 2x
4 100
s = 28. Ans: (28)
20
Decrease in the price of B = (x ) = 0 ⋅ 2x
20. Let Ravi’s salary after the increment be 4x 100
∴ Before the increment it was 3x and the total family income As the increase equals the decrease, the sum of the prices
was 15x. After Ravi’s increment, the family income is 16x. does not change.
Required percentage = ∴ required percentage change = 0% Ans: (0)
Ravi' s new salary 4x
(100%) = (100%) = 25%.
Total family income 16 x 28. Suppose there is a group of quantities. If all are increased /
Choice (D) decreased by the same percentage, their sum will
increase/decrease by the same percentage. Otherwise we
21. As the salaries of the two persons are equal and since the can say that the change in the sum is greater than the least
percentage increase in the salary for one person equals the percentage and less than the greatest percentage.
percentage decrease in the salary for the other, the The salaries of some of the employees are increased by a
increase in the salary and the decrease in the salary must different percentage compared to that of the others.
be equal. ∴ We cannot determine on the percentage change in the
∴ the total salary of the 2 persons does not change. total salary. We can say it has to be between 10% and 20%.
Choice (C) Choice (D)

22. If any quantity becomes n times. 29. Let the initial price of the article be 100
We say it increased by (n − 1) 100%. Decrease in the price = 20
As the population quadrupled, it increases by Price after the decrease = 80
(4 − 1) 100% = 300%. Choice (C) Increase in the price = 16
Price after the increase = 96
23. Let Bala’s marks be 100. ∴ the price effectively decreases by 4% Choice (C)

20 30. Let the initial price of the article be 100


Arun’s mark = 100 + (100 ) = 120 Increase in the price = 10
100
Price after the increase = 110
20
Required % = (100 ) = 16 2 %. Choice (B) Decrease in the price = 11
120 3 Price after the decrease = 9
∴ the price effectively decreases by 1%
24. Let the original price be x
Decrease in the price = 0⋅2x Note: It can be seen that in problems 1 and 2 the
Price after the decrease = 0⋅8x final results are the same although the order of operations vary
∴ in problems on successive percentages the operations
0 ⋅ 2x can be performed in any order as the result does not
% increase = (100 ) = 25%
0 ⋅ 8x depend on the order of the operations. Choice (C)
31. Let the number be x. 42. Total cost price (80) (12·50) = 1000
20 Total profit = (80) (4⋅50) = 360
Decrease in it = x = 0.2x Total selling price = 1360
100
Total selling price of 20 articles = 360
Its new value = 0.8x
Selling price of each of the remaining articles
20
Increase in the new value = (0.8x) = 0.16x 1360 − 360 2
100 = = 16 Choice (B)
Final value of the number = 0.96x which is < x 60 3
∴ The number decreased.
x − 0.96x 43. Let the M.P. be 100
It decreased by (100)% i.e., 4% 30
x First discount (in ) = (100) = 30
Choice (C) 100
Price after this discount = 70
32. Let the first, second and the third numbers be f, s and 100 10
respectively. Second discount (in ) = (70) = 7.
100
40 Price after this discount = 63
f = 100 1 − = 60 Total discount = 37
100
Equivalent single discount percentage = 37%
50 Ans: (37)
s = 100 1− = 50
100 44. Let his C.P. = 100
Required percentage = 60 (100)% = 120%. Choice (D)
25
His M.P (in ) = 100 1+ = 125
33. The salary Raja’s as a percentage of his family’s 100
total income increased by 10 percentage points.
Choice (C) 30
His discount (in ) = (125) = 37.50
100
34. Profit = 36 = 6% of CP His S.P. = 87.50
6 His loss = 12.50
× CP = 36 CP = 600 Ans: (600)
100 12 .5
His loss percentage = (100) %
100
35. Suppose 2 articles have the same cost price and one is sold at = 12.5% Choice (D)
x% profit and the other is sold at y% loss. The overall profit/loss
1 45. Let his C.P. = 100
percentage is given by (difference of x and y)%
2
As x = 20 and y = 10, overall 40
M.P (in ) = 100 1+ = 140
Profit/loss percentage = 5% profit Choice (D) 100

36. Let the selling price be x 12


profit in ( ) = (100) = 12
Cost price = 0⋅9x 100
Profit = 0⋅1x S.P. = 112
0 ⋅ 1x discount = 28.
Profit percentage = (100 ) = 11 1 % Choice (C) discount ( Discount = M.P – S.P)
0 ⋅ 9x 9
28
= (100) % = 20% Ans: (20)
1 140
100 + 33
100 + 20 3
37. Selling price = 50
100 100 46. Let the marked price be x.
Selling price = 400
6 4 1
100 − 11
= 50 = 80 Ans: (80) 9 x = 400
5 3 x = 450 Ans: (450)
100
38. Let the cost price of the item be x
47. Ratio of profits of Ajay and Vijay = 9 : 12 = 3 : 4
Profit = 0⋅2x
x + 0⋅2x = 60 x = 50 3
To gain 30% profit must be 0⋅3x
Ajay’s share = (3500 ) = 1500 Choice (A)
7
∴ selling price must be 1⋅3x = 65 Choice (B)
48. Ratio of profits of Ramesh and Suresh
39. Let the cost price of each in be x = (9000) (10) : (6000) (12) = 5 : 4
Profit made on selling 5 m = cost price of 2 m = 2x 4
Cost price of 5 m = 5x Suresh’s share = (4500 ) = 2000 Choice (D)
9
2
Profit percentage = (100 )% = 40 % Choice (C)
5 6
49. Annual income of Ashok = (12000 ) = 720
100
60 − 50
40. Required percentage = (100 )% = 20% Ans: (720)
50
Choice (C) 50. Market value = 105
12600
41. Profits of the company, dealer and shopkeeper are 10, Number of shares purchased = = 120
15 and 25 respectively. 105
∴ The shopkeeper got the highest profit Choice (C) Annual income of Ajay = (5) (120) = 600 Choice (A)

You might also like